Quantcast
  • Register
PhysicsOverflow is a next-generation academic platform for physicists and astronomers, including a community peer review system and a postgraduate-level discussion forum analogous to MathOverflow.

Welcome to PhysicsOverflow! PhysicsOverflow is an open platform for community peer review and graduate-level Physics discussion.

Please help promote PhysicsOverflow ads elsewhere if you like it.

News

PO is now at the Physics Department of Bielefeld University!

New printer friendly PO pages!

Migration to Bielefeld University was successful!

Please vote for this year's PhysicsOverflow ads!

Please do help out in categorising submissions. Submit a paper to PhysicsOverflow!

... see more

Tools for paper authors

Submit paper
Claim Paper Authorship

Tools for SE users

Search User
Reclaim SE Account
Request Account Merger
Nativise imported posts
Claim post (deleted users)
Import SE post

Users whose questions have been imported from Physics Stack Exchange, Theoretical Physics Stack Exchange, or any other Stack Exchange site are kindly requested to reclaim their account and not to register as a new user.

Public \(\beta\) tools

Report a bug with a feature
Request a new functionality
404 page design
Send feedback

Attributions

(propose a free ad)

Site Statistics

205 submissions , 163 unreviewed
5,047 questions , 2,200 unanswered
5,345 answers , 22,709 comments
1,470 users with positive rep
816 active unimported users
More ...

  Dirac Equation in General Relativity

+ 2 like - 0 dislike
1280 views

Dirac equation for the massless fermions in curved spase time is $γ^ae^μ_aD_μΨ=0$, where $e^μ_a$ are the tetrads. I have to show that Dirac spinors obey the following equation: $$(−D_μD^μ+\frac{1}{4}R)Ψ=0\qquad(1)$$

where $R$ is the Ricci scalar.

I already know that $[D_\mu,D_\nu]A^\rho={{R_{\mu\nu}}^\rho}_\sigma A^\sigma$, but a key point is to know what $[D_\mu,D_\nu]\Psi$ is.

($D_μΨ=∂_μΨ+A^{ab}_μΣ_{ab}$ is the covariant derivative of the spinor field and $Σ_{ab}$ the Lorentz generators involving gamma matrices).

This post imported from StackExchange Physics at 2014-03-22 17:27 (UCT), posted by SE-user Gauge
asked Jan 16, 2013 in Theoretical Physics by Gauge (10 points) [ no revision ]
The right way to solve it is to act with $D_\mu$ from the left on your Dirac GR equation again. Once you do so, you must realize that the new covariant derivative also acts on the tetrads which means that it effectively differentiates the metric and ultimately produces the Ricci scalar term. However, you must realize that the adjective "covariant" means that it has two new connection terms, one from the curved spacetime metric and one from the electromagnetic potential. The latter isn't needed at all for your desired derivation - the gauge field is just kept everywhere as a part of $D_\mu$.

This post imported from StackExchange Physics at 2014-03-22 17:27 (UCT), posted by SE-user Luboš Motl
Related is physics.stackexchange.com/questions/51269/… Are these really distinct? If not, might consider asking mods to merge them.

This post imported from StackExchange Physics at 2014-03-22 17:27 (UCT), posted by SE-user twistor59
Thanks for the comment, but $D_\mu e^\nu_a$ is not zero?

This post imported from StackExchange Physics at 2014-03-22 17:27 (UCT), posted by SE-user Gauge
Aren't the tetrads covariantly constant provided you include both the Christoffel symbols and the spin connection?

This post imported from StackExchange Physics at 2014-03-22 17:27 (UCT), posted by SE-user twistor59
Yes, they are! Can you write explicitly what you mean for $D_\mu e^\nu_a$ and how this can solve the problem... I'm now a bit confused about the proper action of $D_\mu$ on tetrads.

This post imported from StackExchange Physics at 2014-03-22 17:27 (UCT), posted by SE-user Gauge
I would say the cov derivative of the tetrad is something like $ \partial_{\mu}e_{\nu}^a-\Gamma^{\sigma}_{\mu\nu}e_{\sigma}^a+\omega^{ab}_{\mu}e_‌​{\nu b}$ were $\omega^{ab}_{\mu}$ is the spin connection. (Greek is spacetime and latin is tetrad label, also I'm keeping any EM gauge stuff out of here). (PS I'm not claiming to have solved your problem!)

This post imported from StackExchange Physics at 2014-03-22 17:27 (UCT), posted by SE-user twistor59

1 Answer

+ 4 like - 0 dislike

Denoting by $\gamma^a$ the Minkowski space gamma matrices with respect to the Lorentz tetrad $\{e^a\}$, and covariant derivative $D_a$, then the gammas are covariantly constant.

Start with the massless Dirac equation $$ \gamma^{b}D_{b}\Psi = 0$$

Act again with the Dirac operator $$\gamma^{a}D_{a}\gamma^{b}D_{b}\Psi=0 $$ So, since $D$ annihilates $\gamma$ $$\gamma^{a}\gamma^{b}D_{a}D_{b}\Psi = 0 $$ so $$\frac{1}{2}\{\gamma^{a},\gamma^{b}\}D_{a}D_{b}\Psi + \frac{1}{2}\gamma^{a}\gamma^{b}[D_a,D_b]\Psi = 0 \ \ (1) $$ But $$\{\gamma^{a},\gamma^{b}\}=2\eta^{ab} $$ and $$ [D_a,D_b]\Psi = {\mathcal{R}_{ab}}\Psi $$ Where ${\mathcal{R}}_{ab}$ is the spin-curvature (antisymmetric in a and b). ${\mathcal{R}}_{ab}$ satisfies the identity $$ -\gamma^b{\mathcal{R}}_{ab} = {\mathcal{R}}_{ab}\gamma^b = \frac{1}{2}\gamma^b R_{ab}$$ where $R_{ab}$ is the Ricci tensor (in the Lorentz tetrad). so (1) becomes $$ [D^aD_a+\frac{1}{4}\gamma^a\gamma^bR_{ab}]\Psi = 0 $$ i.e. $$ [D^aD_a-\frac{1}{4}R]\Psi = 0 $$

This post imported from StackExchange Physics at 2014-03-22 17:27 (UCT), posted by SE-user twistor59
answered Jan 17, 2013 by twistor59 (2,500 points) [ no revision ]

Your answer

Please use answers only to (at least partly) answer questions. To comment, discuss, or ask for clarification, leave a comment instead.
To mask links under text, please type your text, highlight it, and click the "link" button. You can then enter your link URL.
Please consult the FAQ for as to how to format your post.
This is the answer box; if you want to write a comment instead, please use the 'add comment' button.
Live preview (may slow down editor)   Preview
Your name to display (optional):
Privacy: Your email address will only be used for sending these notifications.
Anti-spam verification:
If you are a human please identify the position of the character covered by the symbol $\varnothing$ in the following word:
p$\hbar$ysicsO$\varnothing$erflow
Then drag the red bullet below over the corresponding character of our banner. When you drop it there, the bullet changes to green (on slow internet connections after a few seconds).
Please complete the anti-spam verification




user contributions licensed under cc by-sa 3.0 with attribution required

Your rights
...